A Fibonacci Sum

Calculus Level 2

The value of the infinite sum

k = 1 F k 2 k = 1 2 + 1 4 + 2 8 + 3 16 + 5 32 + 8 64 + , \sum_{k=1}^\infty \frac{F_k}{2^k} = \frac{1}{2}+\frac{1}{4}+\frac{2}{8}+\frac{3}{16}+\frac{5}{32}+\frac{8}{64}+\ldots,

where F k F_k represents the k t h k^{th} Fibonacci number, can be written as a b \frac{a}{b} , where a a and b b are positive coprime integers. Find a + b a+b .


The answer is 3.

This section requires Javascript.
You are seeing this because something didn't load right. We suggest you, (a) try refreshing the page, (b) enabling javascript if it is disabled on your browser and, finally, (c) loading the non-javascript version of this page . We're sorry about the hassle.

8 solutions

Matt Enlow
Apr 15, 2014

Let x be the desired sum. Then

2 x = 1 + 1 2 + 2 4 + 3 8 + 5 16 + 8 32 + 2x=1+\frac{1}{2}+\frac{2}{4}+\frac{3}{8}+\frac{5}{16}+\frac{8}{32}+\ldots

Subtracting the series that x x represents from the 2 x 2x series above yields

2 x x = 1 + 1 4 + 1 8 + 2 16 + 3 32 + , 2x-x=1+\frac{1}{4}+\frac{1}{8}+\frac{2}{16}+\frac{3}{32}+\ldots,

which is equal to 1 + 1 2 x 1+\frac{1}{2}x . So solving the equation x = 1 + 1 2 x x=1+\frac{1}{2}x for x x yields x = 2 = 2 1 x=2=\frac{2}{1} . Therefore the answer is 2 + 1 = 3 2+1=\boxed{3} .

thats it......so easy....and so beautiful

Max B - 7 years, 1 month ago

I did something similar by expanding F k = F k 1 + F k 2 F_k = F_{k-1} + F_{k-2} and then factoring out powers of 2 2 and changing variables (you have to be careful and set F 0 = 0 F_0 = 0 and F 1 = 1 F_{-1} = 1 ). That leads to x = 1 2 x + 1 4 ( x + 2 ) x = \frac12 x + \frac14 (x+2) .

Patrick Corn - 7 years, 1 month ago

We can also use this :

If we call S the sum of this quantity, so :

S = \sum {k=1}^∞ \frac {F {k}}{2^{k}}

=  \sum_{k=0}^∞ \frac {F_{k}}{2^{k}}  (F_{0}=0)

= \frac 12F_{1} + \sum_{k=2}^∞ \frac {F_{k-1}+F_{k-2}}{2^{k}}

=  \frac {1}{2} + \sum_{k=2}^∞ \frac {F_{k-1}}}{2^{k}} + \sum_{k=2}^∞ \frac {F_{k-2}}{2^{k}}

=  \frac {1}{2} +  \frac {1}{2} \times \sum_{k=0}^∞ \frac {F_{k}}}{2^{k}} + \frac {1}{2^{2}} \times \sum_{k=0}^∞ \frac {F_{k}}{2^{k}}

 = \frac {1}{2} + \frac 12S + \frac 12^{2}S

Then, S = \frac {2}{2^{k} - 2 - 1} = 2 = \frac {2}{1}

a+b = 2+1 = 3

Med sen - 7 years, 1 month ago

but sir I could not understand how this equals to 1+1/2x

ashutosh mahapatra - 7 years, 1 month ago

Log in to reply

this is 1+x/2

Devesh Pathak - 7 years, 1 month ago

Log in to reply

x/2=1/4+1/8+.......from first equation deviding by 4

Devesh Pathak - 7 years, 1 month ago

Oh my lorde I did that exact method and got that exact answer but the problem led me to believe it was a fancy fraction instead of 2/1. My self inflicted torment.

Spock Weakhypercharge - 7 years, 1 month ago

1 is not a prime. I can prove it

Junaid Akhtér - 6 years, 9 months ago

Log in to reply

The numbers 1 and −1 are coprime to every integer, and they are the only integers to be coprime with 0.

Rohit Sharma - 6 years, 4 months ago

Sir, you are changing the infinite series by subtracting and multiplying. Use same method for sum of 1+2+2^2 +2^3.... A=1+2+2^2 +2^3.... 2A = 2+2^2+2^3.... Subtract both equations you will get A=-1

suresh jh - 3 years, 1 month ago

Beautiful Solution!

Fahim Shahriar Shakkhor - 6 years, 8 months ago
Albert Han
Apr 21, 2014

Well since I cannot be beautiful I'll just beLet's see the property of Fibonacci Sum:

F k 1 + F k = F k + 1 F_{k-1}+F_k=F_{k+1}

Then, we can apply this to the sum, then the sums will look like 1 2 + 1 4 + 1 + 1 8 + 1 + 2 16 + 2 + 3 32 + . . . \frac{1}{2} + \frac{1}{4} +\frac{1+1}{8} +\frac {1+2}{16} +\frac{2+3}{32} + ... = 1 2 + 1 4 + 1 8 + 1 8 + 1 16 + 2 16 + 3 32 + 2 32 . . . =\frac{1}{2} + \frac{1}{4} +\frac{1}{8}+\frac{1}{8} +\frac {1}{16}+\frac{2}{16} +\frac{3}{32} + \frac{2}{32} ... = 1 2 + ( 1 4 + 1 8 + 2 16 + 3 32 + . . . ) + ( 1 8 + 1 16 + 2 32 + 3 64 + . . . ) =\frac{1}{2} +(\frac{1}{4} +\frac{1}{8} +\frac{2}{16} +\frac{3}{32}+...) +(\frac{1}{8} +\frac{1}{16} +\frac{2} {32} +\frac{3}{64}+...) Then we can change the second and third term and express as k = 1 F k 2 k = 1 2 + k = 1 F k 2 k + 1 + k = 1 F k 2 k + 2 \sum_{k=1}^\infty \frac{F_k} {2^{k}}=\frac{1}{2} + \sum_{k=1}^\infty \frac{F_k} {2^{k+1}}+\sum_{k=1}^\infty \frac{F_k} {2^{k+2}} k = 1 F k 2 k = 1 2 + 1 2 k = 1 F k 2 k + 1 4 k = 1 F k 2 k \sum_{k=1}^\infty \frac{F_k} {2^{k}}=\frac{1}{2} + \frac{1}{2} \sum_{k=1}^\infty \frac{F_k} {2^{k}}+\frac{1}{4}\sum_{k=1}^\infty \frac{F_k} {2^{k}} 1 4 k = 1 F k 2 k = 1 2 \frac{1}{4}\sum_{k=1}^\infty \frac{F_k} {2^{k}}=\frac{1}{2} k = 1 F k 2 k = 2 1 \sum_{k=1}^\infty \frac{F_k} {2^{k}}=\frac{2}{1} Thus the answer is 3 \boxed{3}

very good :) How can you write solution like this ?

Med sen - 7 years, 1 month ago

Log in to reply

If you are talking about the Latex format, the instruction is in formatting guide, which is below post or comment section.

Albert Han - 7 years, 1 month ago

Thanks Albert.You are really great.

ALINJAR DAN - 7 years, 1 month ago

Oh just realized that I saw comets that had almost identical solution as these. Sorry for repost!

Albert Han - 7 years, 1 month ago
Stefan Stankovic
Apr 21, 2014

The generating function for the Fibonacci numbers is F ( x ) = k = 0 F n x n = x 1 x x 2 F(x)=\sum_{k=0}^{\infty}F_n x^n=\frac{x}{1-x-x^2} and the sum is equal to k = 1 F k ( 1 2 ) k = k = 0 F k ( 1 2 ) k = F ( 1 2 ) = 2 = 1 2 \sum_{k=1}^{\infty}F_k \left (\frac{1}{2}\right )^k=\sum_{k=0}^{\infty}F_k \left (\frac{1}{2}\right )^k=F\left (\frac{1}{2}\right )=2=\frac{1}{2} .

This is incredible use of Discrete Mathematics.

Chitra Singh - 7 years, 1 month ago
Yong See Foo
Aug 10, 2017

Some solutions here assume that the series converges - here's a solution using probabilities to 'avoid' that issue:

Suppose A and B are playing a game consisting of several rounds. They both have a 1 2 \frac{1}{2} chance of winning any round. Suppose the game ends when one of the players wins three consecutive rounds. Let S n S_n be a set containing the sequences of winners of each round such that the game ends in n n rounds, where n 3 n\ge 3 , e.g. S 5 = { A A B B B , A B A A A , B A B B B , B B A A A } S_5 = \{AABBB, ABAAA, BABBB, BBAAA\} . Let T n T_n be a subset of S n S_n containing the sequences that start with two different winners, e.g. T 5 = { A B A A A , B A B B B } T_5 = \{ABAAA, BABBB\} .

Let m 4 m\ge 4 . There is bijection between T m T_m and S m 1 S_{m-1} , by just removing the first player in each sequence of T m T_m (the inverse is to prepend the other player who is not the initial winner for each sequece of S m 1 S_{m-1} to that same sequence). Furthermore, there is a bijection between S m \ T m S_m\backslash T_m and T m 1 T_{m-1} , by removing the first player in each sequence of S m \ T m S_m\backslash T_m (the inverse is to prepend the same player who is the initial winner for each sequence of T m 1 T_{m-1} to that same sequence). Thus we know that S m = T m + S m \ T m = S m 1 + T m 1 |S_m|=|T_m|+|S_m\backslash T_m|=|S_{m-1}|+|T_{m-1}|

Define the Fibonacci sequence by F 0 = 0 , F 1 = 1 , F i = F i 1 + F i 2 F_0=0, F_1=1, F_i=F_{i-1}+F_{i-2} for i 2 i\ge 2 . Now observe that S 3 = 2 , T 3 = 0 |S_3|=2, |T_3|=0 . We claim that S n = 2 F n 2 , T n = 2 F n 3 |S_n|=2F_{n-2},|T_n|=2F_{n-3} . We can prove this by induction, for which the base case n = 3 n=3 is already done. The inductive step is essentially the last equation in the previous paragraph.

The probability that the game ends in n n rounds is therefore S n 2 n = F n 2 2 n 1 \frac{|S_n|}{2^n}=\frac{F_{n-2}}{2^{n-1}} . Since the sum of probabilities is 1 1 , we conclude that k = 1 F k 2 k = n = 3 F n 2 2 n 2 = 2 n = 3 F n 2 2 n 1 = 2 n = 3 probability that game ends in n rounds = 2 \sum_{k=1}^\infty \frac{F_k}{2^k}=\sum_{n=3}^\infty \frac{F_{n-2}}{2^{n-2}}=2\sum_{n=3}^\infty \frac{F_{n-2}}{2^{n-1}}=2\sum_{n=3}^\infty\text{probability that game ends in }n\text{ rounds}=2

Kartik Sharma
Nov 6, 2014

S = 1 2 + 1 4 + 2 8 + 3 16 + . . . . . . . S = \frac{1}{2} + \frac{1}{4} + \frac{2}{8} + \frac{3}{16} + ....... \infty

S 4 = 1 8 + 1 16 + 2 32 + 3 64 + . . . . . . . \frac{S}{4} = \frac{1}{8} + \frac{1}{16} + \frac{2}{32} + \frac{3}{64} + ....... \infty

S S 4 = 1 2 + 1 4 + 1 8 + 2 16 + 3 32 + . . . . S - \frac{S}{4} = \frac{1}{2} + \frac{1}{4} + \frac{1}{8} + \frac{2}{16} + \frac{3}{32} +....\infty

Now for the 4th term of the above series to be a perfect term of GP we need to subtract the above series with S 8 \frac{S}{8} and so on.

S ( S 4 + S 8 + S 16 + . . . . ) = 1 2 + 1 4 + 1 8 + 1 16 + 1 32 + . . . . S - (\frac{S}{4} + \frac{S}{8} + \frac{S}{16} +....\infty) = \frac{1}{2} + \frac{1}{4} + \frac{1}{8} + \frac{1}{16} + \frac{1}{32} +....\infty

Now, we have 2 G.P.s and so just use the GP formula both times to get the answer.

S S 4 1 S 2 = 1 2 1 1 2 S - \frac{\frac{S}{4}}{1-\frac{S}{2}} = \frac{\frac{1}{2}}{1-\frac{1}{2}}

S 2 = 1 \frac{S}{2} = 1

S = 2 S = 2

Med Sen
Apr 21, 2014

We can also use this :

If we call S the sum of this quantity, so :

S = k = 1 F k 2 k = k = 0 F k 2 k ( F 0 = 0 ) = 1 2 F 1 + k = 2 F k 1 + F k 2 2 k = 1 2 + k = 2 F k 1 2 k + k = 2 F k 2 2 k = 1 2 + 1 2 × k = 0 F k 2 k + 1 2 2 × k = 0 F k 2 k = 1 2 + 1 2 S + 1 2 2 S S = \sum{k=1}^\infty \frac {F{k}}{2^{k}} \\ = \sum_{k=0}^\infty \frac {F_{k}}{2^{k}} (F_{0}=0)\\ = \frac 12F_{1} + \sum_{k=2}^\infty \frac {F_{k-1}+F_{k-2}}{2^{k}}\\ = \frac {1}{2} + \sum_{k=2}^\infty \frac {F_{k-1}}{2^{k}} + \sum_{k=2}^\infty \frac {F_{k-2}}{2^{k}}\\ = \frac {1}{2} + \frac {1}{2} \times \sum_{k=0}^\infty \frac {F_{k}}{2^{k}} + \frac {1}{2^{2}} \times \sum_{k=0}^\infty \frac {F_{k}}{2^{k}}\\ = \frac {1}{2} + \frac 12S + \frac 12^{2}S\\

Then, S = 2 2 k 2 1 = 2 = 2 1 S = \frac {2}{2^{k} - 2 - 1} = 2 = \frac {2}{1}

a+b = 2+1 = 3

Note: To type in latex, you need to add brackets around your code: \ ( code \ ) (no spaces). I've edited your solution, so you can take a look at it.

The inifinity symbol is given by \infty. You can get line breaks by using \, or just writing out the different lines in different \ ( \ )

Calvin Lin Staff - 7 years, 1 month ago
Bratch Kroy
Jun 7, 2021

There has been a saying says: "Violence is Art." So i'll give out a violent solution.

As everybody knows, the Fibonacci number has a fomula: F n = 1 5 ( ( 1 + 5 2 ) n ( 1 5 2 ) n ) F_n=\frac{1}{\sqrt{5}}\left(\left(\frac{1+\sqrt{5}}{2}\right)^n-\left(\frac{1-\sqrt{5}}{2}\right)^n\right)

So do a little deform:

n = 1 F n 2 n = n = 1 1 5 ( ( 1 + 5 2 ) n ( 1 5 2 ) n ) 2 n = 1 5 n = 0 ( 1 + 5 ) n ( 1 5 ) n 4 n = 1 5 ( n = 0 ( 1 + 5 4 ) n n = 0 ( 1 + 5 4 ) n ) \begin{aligned} \sum_{n=1}^{\infty}\frac{F_n}{2^n}&=\sum_{n=1}{\infty}\frac{\frac{1}{\sqrt{5}}\left(\left(\frac{1+\sqrt{5}}{2}\right)^n-\left(\frac{1-\sqrt{5}}{2}\right)^n\right)}{2^n}\\ &=\frac{1}{\sqrt{5}}\sum_{n=0}^{\infty}\frac{(1+\sqrt{5})^n-(1-\sqrt{5})^n}{4^n}\\ &=\frac{1}{\sqrt{5}}\left(\sum_{n=0}^{\infty}\left(\frac{1+\sqrt{5}}{4}\right)^n-\sum_{n=0}^{\infty}\left(\frac{1+\sqrt{5}}{4}\right)^n\right) \end{aligned}

and it is a normal geometric sequence. Easy to know that it is 2, which is 2 1 \frac{2}{1}

so the answer is 1 + 2 = 3 1+2=3

Brock Brown
Dec 28, 2014

I used a little bit of Python to get to my answer.

 1
 2
 3
 4
 5
 6
 7
 8
 9
10
11
12
13
14
from fractions import Fraction as frac
memo = {1:1,2:1}
def fib(x):
    if x in memo:
        return memo[x]
    memo[x] = fib(x-1) + fib(x-2)
    return memo[x]
total = 0
k = 1
infinity = 1000
while k <= infinity:
    total += frac(fib(k),2**k)
    k += 1
print "Answer:",float(total)

0 pending reports

×

Problem Loading...

Note Loading...

Set Loading...